6 linear PDE for only 3 unknowns? The 2019 Stack Overflow Developer Survey Results Are In Announcing the arrival of Valued Associate #679: Cesar Manara Planned maintenance scheduled April 17/18, 2019 at 00:00UTC (8:00pm US/Eastern)How to solve this degenerate parabolic euqationProving the uniqueness of a PDE's solutionHow to solve this PDE $partial_t u - Delta u + cu + dcdot nabla u = f$ with Dirichlet boundary conditions?Crank-Nicolson for quadratic PDEMatrix for discretized PDEUsing Fourier sine transform to solve PDEAn estimate for a 1d hyperbolic PDESplitting a PDE into two subdomains: Regularity conditions at the interface point(s)Two PDE for one unknown?Two PDE for one matrix-valued unknown?

Why did all the guest students take carriages to the Yule Ball?

Can a 1st-level character have an ability score above 18?

What aspect of planet Earth must be changed to prevent the industrial revolution?

Road tyres vs "Street" tyres for charity ride on MTB Tandem

How do you keep chess fun when your opponent constantly beats you?

Typeface like Times New Roman but with "tied" percent sign

How is simplicity better than precision and clarity in prose?

Mortgage adviser recommends a longer term than necessary combined with overpayments

Is it ok to offer lower paid work as a trial period before negotiating for a full-time job?

Why can't wing-mounted spoilers be used to steepen approaches?

How to split my screen on my Macbook Air?

Who or what is the being for whom Being is a question for Heidegger?

Would an alien lifeform be able to achieve space travel if lacking in vision?

Why can't devices on different VLANs, but on the same subnet, communicate?

Take groceries in checked luggage

Why is superheterodyning better than direct conversion?

How to test the equality of two Pearson correlation coefficients computed from the same sample?

Does Parliament need to approve the new Brexit delay to 31 October 2019?

What was the last x86 CPU that did not have the x87 floating-point unit built in?

Didn't get enough time to take a Coding Test - what to do now?

How did the audience guess the pentatonic scale in Bobby McFerrin's presentation?

system() function string length limit

Cooking pasta in a water boiler

Can smartphones with the same camera sensor have different image quality?



6 linear PDE for only 3 unknowns?



The 2019 Stack Overflow Developer Survey Results Are In
Announcing the arrival of Valued Associate #679: Cesar Manara
Planned maintenance scheduled April 17/18, 2019 at 00:00UTC (8:00pm US/Eastern)How to solve this degenerate parabolic euqationProving the uniqueness of a PDE's solutionHow to solve this PDE $partial_t u - Delta u + cu + dcdot nabla u = f$ with Dirichlet boundary conditions?Crank-Nicolson for quadratic PDEMatrix for discretized PDEUsing Fourier sine transform to solve PDEAn estimate for a 1d hyperbolic PDESplitting a PDE into two subdomains: Regularity conditions at the interface point(s)Two PDE for one unknown?Two PDE for one matrix-valued unknown?










5












$begingroup$


Let $x in (0,L)$, $t in (0,T)$, and let $u_0 = u_0(x) in mathbbR^3$, $g=g(t) in mathbbR^3$, $P = P(x,t) in mathbbR^3$ and $Q = Q(x,t) in mathbbR^3$ be continuously differentiable functions.



Denote by $hatP, hatQ$ the matrices
beginalign*
hatP = beginbmatrix
0 & -P_3 & P_2 \
Q_3 & 0 & -P_1 \
-P_2 & P_1 & 0
endbmatrix, qquad
hatQ = beginbmatrix
0 & -Q_3 & Q_2 \
Q_3 & 0 & -Q_1 \
-Q_2 & Q_1 & 0
endbmatrix.
endalign*



My question is:




Assuming that the following four conditions hold:



Condition 1:
beginalign*
partial_x P - partial_t Q = hatPQ qquad textin (0,L) times (0,T),
endalign*

Condition 2:
beginalign*
u_0(0) = g(0),
endalign*

Condition 3:
beginalign*
u_0' = - hatQu_0 qquad textfor x in (0,L),
endalign*

Condition 4:
beginalign*
g' = - hatPg qquad textfor t in (0,T),
endalign*

is there a solution $u = u(x,t) in mathbbR^3$ to
beginalign*
begincases
partial_t u = -hatPu &textin (0,L)times(0,T) \
partial_x u = - hatQu &textin (0,L)times(0,T) \
u(x,0) = u_0(x) & textfor x in (0,L)\
u(0,t) = g(t) & textfor t in (0,T)
endcases
endalign*

or not?




In the above, $f'$, $partial_t f$a and $partial_x f$ denote the derivative, partial time derivative and partial space derivative respectively; and $P_k$, $Q_k$, $u_k$, $u_0k$ and $g_k$ denote the components of $P$, $Q$, $u$, $u_0$ and $g$ respectively.



My question with all terms developed rewrites as follows. Assuming that the following four conditions hold:



Condition 1:
beginalign*
begincases
partial_x P_1 - partial_t Q_1 = P_2 Q_3 - P_3 Q_2\
partial_x P_2 - partial_t Q_2 = P_3 Q_1 - P_1 Q_3\
partial_x P_3 - partial_t Q_3 = P_1 Q_2 - P_2 Q_1
endcases qquad textin (0,L) times (0,T),
endalign*

Condition 2:
beginalign*
u_0(0) = g(0),
endalign*

Condition 3:
beginalign*
begincases
u_01' = Q_3(cdot,0) u_02 - Q_2(cdot,0) u_03 \
u_02' = -Q_3(cdot,0) u_01 + Q_1(cdot,0) u_03 \
u_03' = Q_2(cdot,0) u_01 - Q_1(cdot,0) u_02
endcases qquad textfor x in (0,L),
endalign*

Condition 4:
beginalign*
begincases
g_1' &= P_3(0,cdot) g_2 - P_2(0,cdot) g_3 \
g_2' &= -P_3(0,cdot) g_1 + P_1(0,cdot) g_3 \
g_3' &= P_2(0,cdot) g_1 - P_1(0,cdot) g_2
endcases qquad textfor t in (0,T),
endalign*

is there a solution $u = u(x,t) in mathbbR^3$ to the problem:
beginalign*
begincases
partial_t u_1 = P_3 u_2 - P_2 u_3 &textin (0,L)times(0,T) \
partial_t u_2 = -P_3 u_1 + P_1 u_3 &textin (0,L)times(0,T) \
partial_t u_3 = P_2 u_1 - P_1 u_2 & textin (0,L)times(0,T) \
partial_x u_1 = Q_3 u_2 - Q_2 u_3 & textin (0,L)times(0,T) \
partial_x u_2 = -Q_3 u_1 + Q_1 u_3 &textin (0,L)times(0,T) \
partial_x u_3 = Q_2 u_1 - Q_1 u_2 & textin (0,L)times(0,T) \
u(x,0) = u_0(x) & textfor x in (0,L)\
u(0,t) = g(t) & textfor t in (0,T).
endcases
endalign*



What I started. I started reasoning the following way. For $x in (0,L)$ fixed, a function of the form
beginalign*
u_1(x,t) &= u_01(x) + int_0^t P_3(x,s)u_2(x,s) - P_2(x,s)u_3(x,s)ds\
u_2(x,t) &= u_02(x) + int_0^t -P_3(x,s)u_1(x,s) + P_1(x,s)u_3(x,s)ds\
u_3(x,t) &= u_03(x) + int_0^t P_2(x,s)u_1(x,s) - P_1(x,s)u_2(x,s)ds,
endalign*

satisfies the initial value problem involving time derivatives, with $u(cdot, 0) = u_0$, while for $t in (0,T)$ fixed, a solution of the form
beginalign*
beginaligned
u_1(x,t) &= g_1(t) + int_0^x Q_3(xi,t)u_2(xi,t) - Q_2(xi, t)u_3(xi, t)dxi \
u_2(x,t) &= g_2(t) + int_0^x -Q_3(xi, t)u_1(xi, t) + Q_1(xi, t) u_3(xi, t)dxi\
u_3(x,t) &= g_3(t) + int_0^x Q_2(xi, t)u_1(xi, t) - Q_1(xi, t)u_2(xi, t)dxi.
endaligned
endalign*

satisfies the initial value problem involving space derivatives, with $u(0, cdot) = g$. I wanted to show, using the four conditions, that both expressions for the solution coincide. Using conditions 3 and 4 we can rewrite both expressions as
beginalign*
u_1(x,t) &= u_01(0) + int_0^x Q_3(xi, 0)u_02(xi) - Q_2(xi, 0)u_03(xi)dxi \
&quad + int_0^t P_3(0,s)g_2(s) - P_2(0,s)g_3(s)ds + int_0^t int_0^x partial_x (P_3 u_2 - P_2 u_3 )(xi, s)dxi ds\
u_2(x,t) &= u_02(0) + int_0^x -Q_3(xi, 0)u_01 + Q_1(xi, 0)u_03(xi)dxi \
&quad + int_0^t -P_3(0,s)g_1(s) + P_1(0,s)g_3(s)ds + int_0^t int_0^x partial_x(-P_3 u_1 + P_1 u_3)(xi, s)ds\
u_3(x,t) &= u_03(0) + int_0^x Q_2(xi, 0)u_01(xi) - Q_1(xi, 0)u_02(xi)\
&quad + int_0^t P_2(0,s)g_1(s) - P_1(0,s)g_2(s)ds + int_0^t int_0^x partial_x (P_2 u_1 - P_1 u_2)(xi,s)dxi ds,
endalign*

and
beginalign*
u_1(x,t) &= g_1(0) + int_0^t P_3(0,s)g_2(s) - P_2(0,s)g_3(s) ds \
&quad + int_0^x Q_3(xi,0)u_02(xi) - Q_2(xi, 0)u_03(xi)dxi + int_0^x int_0^t partial_t ( Q_3 u_2 - Q_2 u_3)(xi, s)dsdxi\
u_2(x,t) &= g_2(0) + int_0^t -P_3(0,s)g_1(s) + P_1(0,s)g_3(s) ds\
&quad + int_0^x -Q_3(xi, 0)u_01(xi) + Q_1(xi, 0) u_03(xi)dxi + int_0^x int_0^t partial_t (-Q_3 u_1 + Q_1 u_3)(xi,s)dsdxi\
u_3(x,t) &= g_3(0) + int_0^t P_2(0,s)g_1(s) - P_1(0,s)g_2(s) ds \
&quad + int_0^x Q_2(xi, 0)u_01(xi) - Q_1(xi, 0)u_02(xi)dxi + int_0^x int_0^t partial_t ( Q_2 u_1 - Q_1 u_2)(xi, s)ds dxi
endalign*

respectively. It seems that both expressions would coincide if the following equalities hold:
beginalign*
partial_x (P_3 u_2 - P_2 u_3 ) &= partial_t ( Q_3 u_2 - Q_2 u_3)\
partial_x(-P_3 u_1 + P_1 u_3) &= partial_t (-Q_3 u_1 + Q_1 u_3)\
partial_x (P_2 u_1 - P_1 u_2) &= partial_t ( Q_2 u_1 - Q_1 u_2).
endalign*

To show that these equalities hold, I would like to use condition 1. However, I do not know how to argue from there.



Any suggestion of how to proceed from here, any alternative way of reasoning, or explanation of why there would be no solution, would be welcome. Thank you.



This question is also posed on MathOverflow: https://mathoverflow.net/questions/326280/6-linear-pde-for-only-3-unknowns.










share|cite|improve this question











$endgroup$
















    5












    $begingroup$


    Let $x in (0,L)$, $t in (0,T)$, and let $u_0 = u_0(x) in mathbbR^3$, $g=g(t) in mathbbR^3$, $P = P(x,t) in mathbbR^3$ and $Q = Q(x,t) in mathbbR^3$ be continuously differentiable functions.



    Denote by $hatP, hatQ$ the matrices
    beginalign*
    hatP = beginbmatrix
    0 & -P_3 & P_2 \
    Q_3 & 0 & -P_1 \
    -P_2 & P_1 & 0
    endbmatrix, qquad
    hatQ = beginbmatrix
    0 & -Q_3 & Q_2 \
    Q_3 & 0 & -Q_1 \
    -Q_2 & Q_1 & 0
    endbmatrix.
    endalign*



    My question is:




    Assuming that the following four conditions hold:



    Condition 1:
    beginalign*
    partial_x P - partial_t Q = hatPQ qquad textin (0,L) times (0,T),
    endalign*

    Condition 2:
    beginalign*
    u_0(0) = g(0),
    endalign*

    Condition 3:
    beginalign*
    u_0' = - hatQu_0 qquad textfor x in (0,L),
    endalign*

    Condition 4:
    beginalign*
    g' = - hatPg qquad textfor t in (0,T),
    endalign*

    is there a solution $u = u(x,t) in mathbbR^3$ to
    beginalign*
    begincases
    partial_t u = -hatPu &textin (0,L)times(0,T) \
    partial_x u = - hatQu &textin (0,L)times(0,T) \
    u(x,0) = u_0(x) & textfor x in (0,L)\
    u(0,t) = g(t) & textfor t in (0,T)
    endcases
    endalign*

    or not?




    In the above, $f'$, $partial_t f$a and $partial_x f$ denote the derivative, partial time derivative and partial space derivative respectively; and $P_k$, $Q_k$, $u_k$, $u_0k$ and $g_k$ denote the components of $P$, $Q$, $u$, $u_0$ and $g$ respectively.



    My question with all terms developed rewrites as follows. Assuming that the following four conditions hold:



    Condition 1:
    beginalign*
    begincases
    partial_x P_1 - partial_t Q_1 = P_2 Q_3 - P_3 Q_2\
    partial_x P_2 - partial_t Q_2 = P_3 Q_1 - P_1 Q_3\
    partial_x P_3 - partial_t Q_3 = P_1 Q_2 - P_2 Q_1
    endcases qquad textin (0,L) times (0,T),
    endalign*

    Condition 2:
    beginalign*
    u_0(0) = g(0),
    endalign*

    Condition 3:
    beginalign*
    begincases
    u_01' = Q_3(cdot,0) u_02 - Q_2(cdot,0) u_03 \
    u_02' = -Q_3(cdot,0) u_01 + Q_1(cdot,0) u_03 \
    u_03' = Q_2(cdot,0) u_01 - Q_1(cdot,0) u_02
    endcases qquad textfor x in (0,L),
    endalign*

    Condition 4:
    beginalign*
    begincases
    g_1' &= P_3(0,cdot) g_2 - P_2(0,cdot) g_3 \
    g_2' &= -P_3(0,cdot) g_1 + P_1(0,cdot) g_3 \
    g_3' &= P_2(0,cdot) g_1 - P_1(0,cdot) g_2
    endcases qquad textfor t in (0,T),
    endalign*

    is there a solution $u = u(x,t) in mathbbR^3$ to the problem:
    beginalign*
    begincases
    partial_t u_1 = P_3 u_2 - P_2 u_3 &textin (0,L)times(0,T) \
    partial_t u_2 = -P_3 u_1 + P_1 u_3 &textin (0,L)times(0,T) \
    partial_t u_3 = P_2 u_1 - P_1 u_2 & textin (0,L)times(0,T) \
    partial_x u_1 = Q_3 u_2 - Q_2 u_3 & textin (0,L)times(0,T) \
    partial_x u_2 = -Q_3 u_1 + Q_1 u_3 &textin (0,L)times(0,T) \
    partial_x u_3 = Q_2 u_1 - Q_1 u_2 & textin (0,L)times(0,T) \
    u(x,0) = u_0(x) & textfor x in (0,L)\
    u(0,t) = g(t) & textfor t in (0,T).
    endcases
    endalign*



    What I started. I started reasoning the following way. For $x in (0,L)$ fixed, a function of the form
    beginalign*
    u_1(x,t) &= u_01(x) + int_0^t P_3(x,s)u_2(x,s) - P_2(x,s)u_3(x,s)ds\
    u_2(x,t) &= u_02(x) + int_0^t -P_3(x,s)u_1(x,s) + P_1(x,s)u_3(x,s)ds\
    u_3(x,t) &= u_03(x) + int_0^t P_2(x,s)u_1(x,s) - P_1(x,s)u_2(x,s)ds,
    endalign*

    satisfies the initial value problem involving time derivatives, with $u(cdot, 0) = u_0$, while for $t in (0,T)$ fixed, a solution of the form
    beginalign*
    beginaligned
    u_1(x,t) &= g_1(t) + int_0^x Q_3(xi,t)u_2(xi,t) - Q_2(xi, t)u_3(xi, t)dxi \
    u_2(x,t) &= g_2(t) + int_0^x -Q_3(xi, t)u_1(xi, t) + Q_1(xi, t) u_3(xi, t)dxi\
    u_3(x,t) &= g_3(t) + int_0^x Q_2(xi, t)u_1(xi, t) - Q_1(xi, t)u_2(xi, t)dxi.
    endaligned
    endalign*

    satisfies the initial value problem involving space derivatives, with $u(0, cdot) = g$. I wanted to show, using the four conditions, that both expressions for the solution coincide. Using conditions 3 and 4 we can rewrite both expressions as
    beginalign*
    u_1(x,t) &= u_01(0) + int_0^x Q_3(xi, 0)u_02(xi) - Q_2(xi, 0)u_03(xi)dxi \
    &quad + int_0^t P_3(0,s)g_2(s) - P_2(0,s)g_3(s)ds + int_0^t int_0^x partial_x (P_3 u_2 - P_2 u_3 )(xi, s)dxi ds\
    u_2(x,t) &= u_02(0) + int_0^x -Q_3(xi, 0)u_01 + Q_1(xi, 0)u_03(xi)dxi \
    &quad + int_0^t -P_3(0,s)g_1(s) + P_1(0,s)g_3(s)ds + int_0^t int_0^x partial_x(-P_3 u_1 + P_1 u_3)(xi, s)ds\
    u_3(x,t) &= u_03(0) + int_0^x Q_2(xi, 0)u_01(xi) - Q_1(xi, 0)u_02(xi)\
    &quad + int_0^t P_2(0,s)g_1(s) - P_1(0,s)g_2(s)ds + int_0^t int_0^x partial_x (P_2 u_1 - P_1 u_2)(xi,s)dxi ds,
    endalign*

    and
    beginalign*
    u_1(x,t) &= g_1(0) + int_0^t P_3(0,s)g_2(s) - P_2(0,s)g_3(s) ds \
    &quad + int_0^x Q_3(xi,0)u_02(xi) - Q_2(xi, 0)u_03(xi)dxi + int_0^x int_0^t partial_t ( Q_3 u_2 - Q_2 u_3)(xi, s)dsdxi\
    u_2(x,t) &= g_2(0) + int_0^t -P_3(0,s)g_1(s) + P_1(0,s)g_3(s) ds\
    &quad + int_0^x -Q_3(xi, 0)u_01(xi) + Q_1(xi, 0) u_03(xi)dxi + int_0^x int_0^t partial_t (-Q_3 u_1 + Q_1 u_3)(xi,s)dsdxi\
    u_3(x,t) &= g_3(0) + int_0^t P_2(0,s)g_1(s) - P_1(0,s)g_2(s) ds \
    &quad + int_0^x Q_2(xi, 0)u_01(xi) - Q_1(xi, 0)u_02(xi)dxi + int_0^x int_0^t partial_t ( Q_2 u_1 - Q_1 u_2)(xi, s)ds dxi
    endalign*

    respectively. It seems that both expressions would coincide if the following equalities hold:
    beginalign*
    partial_x (P_3 u_2 - P_2 u_3 ) &= partial_t ( Q_3 u_2 - Q_2 u_3)\
    partial_x(-P_3 u_1 + P_1 u_3) &= partial_t (-Q_3 u_1 + Q_1 u_3)\
    partial_x (P_2 u_1 - P_1 u_2) &= partial_t ( Q_2 u_1 - Q_1 u_2).
    endalign*

    To show that these equalities hold, I would like to use condition 1. However, I do not know how to argue from there.



    Any suggestion of how to proceed from here, any alternative way of reasoning, or explanation of why there would be no solution, would be welcome. Thank you.



    This question is also posed on MathOverflow: https://mathoverflow.net/questions/326280/6-linear-pde-for-only-3-unknowns.










    share|cite|improve this question











    $endgroup$














      5












      5








      5





      $begingroup$


      Let $x in (0,L)$, $t in (0,T)$, and let $u_0 = u_0(x) in mathbbR^3$, $g=g(t) in mathbbR^3$, $P = P(x,t) in mathbbR^3$ and $Q = Q(x,t) in mathbbR^3$ be continuously differentiable functions.



      Denote by $hatP, hatQ$ the matrices
      beginalign*
      hatP = beginbmatrix
      0 & -P_3 & P_2 \
      Q_3 & 0 & -P_1 \
      -P_2 & P_1 & 0
      endbmatrix, qquad
      hatQ = beginbmatrix
      0 & -Q_3 & Q_2 \
      Q_3 & 0 & -Q_1 \
      -Q_2 & Q_1 & 0
      endbmatrix.
      endalign*



      My question is:




      Assuming that the following four conditions hold:



      Condition 1:
      beginalign*
      partial_x P - partial_t Q = hatPQ qquad textin (0,L) times (0,T),
      endalign*

      Condition 2:
      beginalign*
      u_0(0) = g(0),
      endalign*

      Condition 3:
      beginalign*
      u_0' = - hatQu_0 qquad textfor x in (0,L),
      endalign*

      Condition 4:
      beginalign*
      g' = - hatPg qquad textfor t in (0,T),
      endalign*

      is there a solution $u = u(x,t) in mathbbR^3$ to
      beginalign*
      begincases
      partial_t u = -hatPu &textin (0,L)times(0,T) \
      partial_x u = - hatQu &textin (0,L)times(0,T) \
      u(x,0) = u_0(x) & textfor x in (0,L)\
      u(0,t) = g(t) & textfor t in (0,T)
      endcases
      endalign*

      or not?




      In the above, $f'$, $partial_t f$a and $partial_x f$ denote the derivative, partial time derivative and partial space derivative respectively; and $P_k$, $Q_k$, $u_k$, $u_0k$ and $g_k$ denote the components of $P$, $Q$, $u$, $u_0$ and $g$ respectively.



      My question with all terms developed rewrites as follows. Assuming that the following four conditions hold:



      Condition 1:
      beginalign*
      begincases
      partial_x P_1 - partial_t Q_1 = P_2 Q_3 - P_3 Q_2\
      partial_x P_2 - partial_t Q_2 = P_3 Q_1 - P_1 Q_3\
      partial_x P_3 - partial_t Q_3 = P_1 Q_2 - P_2 Q_1
      endcases qquad textin (0,L) times (0,T),
      endalign*

      Condition 2:
      beginalign*
      u_0(0) = g(0),
      endalign*

      Condition 3:
      beginalign*
      begincases
      u_01' = Q_3(cdot,0) u_02 - Q_2(cdot,0) u_03 \
      u_02' = -Q_3(cdot,0) u_01 + Q_1(cdot,0) u_03 \
      u_03' = Q_2(cdot,0) u_01 - Q_1(cdot,0) u_02
      endcases qquad textfor x in (0,L),
      endalign*

      Condition 4:
      beginalign*
      begincases
      g_1' &= P_3(0,cdot) g_2 - P_2(0,cdot) g_3 \
      g_2' &= -P_3(0,cdot) g_1 + P_1(0,cdot) g_3 \
      g_3' &= P_2(0,cdot) g_1 - P_1(0,cdot) g_2
      endcases qquad textfor t in (0,T),
      endalign*

      is there a solution $u = u(x,t) in mathbbR^3$ to the problem:
      beginalign*
      begincases
      partial_t u_1 = P_3 u_2 - P_2 u_3 &textin (0,L)times(0,T) \
      partial_t u_2 = -P_3 u_1 + P_1 u_3 &textin (0,L)times(0,T) \
      partial_t u_3 = P_2 u_1 - P_1 u_2 & textin (0,L)times(0,T) \
      partial_x u_1 = Q_3 u_2 - Q_2 u_3 & textin (0,L)times(0,T) \
      partial_x u_2 = -Q_3 u_1 + Q_1 u_3 &textin (0,L)times(0,T) \
      partial_x u_3 = Q_2 u_1 - Q_1 u_2 & textin (0,L)times(0,T) \
      u(x,0) = u_0(x) & textfor x in (0,L)\
      u(0,t) = g(t) & textfor t in (0,T).
      endcases
      endalign*



      What I started. I started reasoning the following way. For $x in (0,L)$ fixed, a function of the form
      beginalign*
      u_1(x,t) &= u_01(x) + int_0^t P_3(x,s)u_2(x,s) - P_2(x,s)u_3(x,s)ds\
      u_2(x,t) &= u_02(x) + int_0^t -P_3(x,s)u_1(x,s) + P_1(x,s)u_3(x,s)ds\
      u_3(x,t) &= u_03(x) + int_0^t P_2(x,s)u_1(x,s) - P_1(x,s)u_2(x,s)ds,
      endalign*

      satisfies the initial value problem involving time derivatives, with $u(cdot, 0) = u_0$, while for $t in (0,T)$ fixed, a solution of the form
      beginalign*
      beginaligned
      u_1(x,t) &= g_1(t) + int_0^x Q_3(xi,t)u_2(xi,t) - Q_2(xi, t)u_3(xi, t)dxi \
      u_2(x,t) &= g_2(t) + int_0^x -Q_3(xi, t)u_1(xi, t) + Q_1(xi, t) u_3(xi, t)dxi\
      u_3(x,t) &= g_3(t) + int_0^x Q_2(xi, t)u_1(xi, t) - Q_1(xi, t)u_2(xi, t)dxi.
      endaligned
      endalign*

      satisfies the initial value problem involving space derivatives, with $u(0, cdot) = g$. I wanted to show, using the four conditions, that both expressions for the solution coincide. Using conditions 3 and 4 we can rewrite both expressions as
      beginalign*
      u_1(x,t) &= u_01(0) + int_0^x Q_3(xi, 0)u_02(xi) - Q_2(xi, 0)u_03(xi)dxi \
      &quad + int_0^t P_3(0,s)g_2(s) - P_2(0,s)g_3(s)ds + int_0^t int_0^x partial_x (P_3 u_2 - P_2 u_3 )(xi, s)dxi ds\
      u_2(x,t) &= u_02(0) + int_0^x -Q_3(xi, 0)u_01 + Q_1(xi, 0)u_03(xi)dxi \
      &quad + int_0^t -P_3(0,s)g_1(s) + P_1(0,s)g_3(s)ds + int_0^t int_0^x partial_x(-P_3 u_1 + P_1 u_3)(xi, s)ds\
      u_3(x,t) &= u_03(0) + int_0^x Q_2(xi, 0)u_01(xi) - Q_1(xi, 0)u_02(xi)\
      &quad + int_0^t P_2(0,s)g_1(s) - P_1(0,s)g_2(s)ds + int_0^t int_0^x partial_x (P_2 u_1 - P_1 u_2)(xi,s)dxi ds,
      endalign*

      and
      beginalign*
      u_1(x,t) &= g_1(0) + int_0^t P_3(0,s)g_2(s) - P_2(0,s)g_3(s) ds \
      &quad + int_0^x Q_3(xi,0)u_02(xi) - Q_2(xi, 0)u_03(xi)dxi + int_0^x int_0^t partial_t ( Q_3 u_2 - Q_2 u_3)(xi, s)dsdxi\
      u_2(x,t) &= g_2(0) + int_0^t -P_3(0,s)g_1(s) + P_1(0,s)g_3(s) ds\
      &quad + int_0^x -Q_3(xi, 0)u_01(xi) + Q_1(xi, 0) u_03(xi)dxi + int_0^x int_0^t partial_t (-Q_3 u_1 + Q_1 u_3)(xi,s)dsdxi\
      u_3(x,t) &= g_3(0) + int_0^t P_2(0,s)g_1(s) - P_1(0,s)g_2(s) ds \
      &quad + int_0^x Q_2(xi, 0)u_01(xi) - Q_1(xi, 0)u_02(xi)dxi + int_0^x int_0^t partial_t ( Q_2 u_1 - Q_1 u_2)(xi, s)ds dxi
      endalign*

      respectively. It seems that both expressions would coincide if the following equalities hold:
      beginalign*
      partial_x (P_3 u_2 - P_2 u_3 ) &= partial_t ( Q_3 u_2 - Q_2 u_3)\
      partial_x(-P_3 u_1 + P_1 u_3) &= partial_t (-Q_3 u_1 + Q_1 u_3)\
      partial_x (P_2 u_1 - P_1 u_2) &= partial_t ( Q_2 u_1 - Q_1 u_2).
      endalign*

      To show that these equalities hold, I would like to use condition 1. However, I do not know how to argue from there.



      Any suggestion of how to proceed from here, any alternative way of reasoning, or explanation of why there would be no solution, would be welcome. Thank you.



      This question is also posed on MathOverflow: https://mathoverflow.net/questions/326280/6-linear-pde-for-only-3-unknowns.










      share|cite|improve this question











      $endgroup$




      Let $x in (0,L)$, $t in (0,T)$, and let $u_0 = u_0(x) in mathbbR^3$, $g=g(t) in mathbbR^3$, $P = P(x,t) in mathbbR^3$ and $Q = Q(x,t) in mathbbR^3$ be continuously differentiable functions.



      Denote by $hatP, hatQ$ the matrices
      beginalign*
      hatP = beginbmatrix
      0 & -P_3 & P_2 \
      Q_3 & 0 & -P_1 \
      -P_2 & P_1 & 0
      endbmatrix, qquad
      hatQ = beginbmatrix
      0 & -Q_3 & Q_2 \
      Q_3 & 0 & -Q_1 \
      -Q_2 & Q_1 & 0
      endbmatrix.
      endalign*



      My question is:




      Assuming that the following four conditions hold:



      Condition 1:
      beginalign*
      partial_x P - partial_t Q = hatPQ qquad textin (0,L) times (0,T),
      endalign*

      Condition 2:
      beginalign*
      u_0(0) = g(0),
      endalign*

      Condition 3:
      beginalign*
      u_0' = - hatQu_0 qquad textfor x in (0,L),
      endalign*

      Condition 4:
      beginalign*
      g' = - hatPg qquad textfor t in (0,T),
      endalign*

      is there a solution $u = u(x,t) in mathbbR^3$ to
      beginalign*
      begincases
      partial_t u = -hatPu &textin (0,L)times(0,T) \
      partial_x u = - hatQu &textin (0,L)times(0,T) \
      u(x,0) = u_0(x) & textfor x in (0,L)\
      u(0,t) = g(t) & textfor t in (0,T)
      endcases
      endalign*

      or not?




      In the above, $f'$, $partial_t f$a and $partial_x f$ denote the derivative, partial time derivative and partial space derivative respectively; and $P_k$, $Q_k$, $u_k$, $u_0k$ and $g_k$ denote the components of $P$, $Q$, $u$, $u_0$ and $g$ respectively.



      My question with all terms developed rewrites as follows. Assuming that the following four conditions hold:



      Condition 1:
      beginalign*
      begincases
      partial_x P_1 - partial_t Q_1 = P_2 Q_3 - P_3 Q_2\
      partial_x P_2 - partial_t Q_2 = P_3 Q_1 - P_1 Q_3\
      partial_x P_3 - partial_t Q_3 = P_1 Q_2 - P_2 Q_1
      endcases qquad textin (0,L) times (0,T),
      endalign*

      Condition 2:
      beginalign*
      u_0(0) = g(0),
      endalign*

      Condition 3:
      beginalign*
      begincases
      u_01' = Q_3(cdot,0) u_02 - Q_2(cdot,0) u_03 \
      u_02' = -Q_3(cdot,0) u_01 + Q_1(cdot,0) u_03 \
      u_03' = Q_2(cdot,0) u_01 - Q_1(cdot,0) u_02
      endcases qquad textfor x in (0,L),
      endalign*

      Condition 4:
      beginalign*
      begincases
      g_1' &= P_3(0,cdot) g_2 - P_2(0,cdot) g_3 \
      g_2' &= -P_3(0,cdot) g_1 + P_1(0,cdot) g_3 \
      g_3' &= P_2(0,cdot) g_1 - P_1(0,cdot) g_2
      endcases qquad textfor t in (0,T),
      endalign*

      is there a solution $u = u(x,t) in mathbbR^3$ to the problem:
      beginalign*
      begincases
      partial_t u_1 = P_3 u_2 - P_2 u_3 &textin (0,L)times(0,T) \
      partial_t u_2 = -P_3 u_1 + P_1 u_3 &textin (0,L)times(0,T) \
      partial_t u_3 = P_2 u_1 - P_1 u_2 & textin (0,L)times(0,T) \
      partial_x u_1 = Q_3 u_2 - Q_2 u_3 & textin (0,L)times(0,T) \
      partial_x u_2 = -Q_3 u_1 + Q_1 u_3 &textin (0,L)times(0,T) \
      partial_x u_3 = Q_2 u_1 - Q_1 u_2 & textin (0,L)times(0,T) \
      u(x,0) = u_0(x) & textfor x in (0,L)\
      u(0,t) = g(t) & textfor t in (0,T).
      endcases
      endalign*



      What I started. I started reasoning the following way. For $x in (0,L)$ fixed, a function of the form
      beginalign*
      u_1(x,t) &= u_01(x) + int_0^t P_3(x,s)u_2(x,s) - P_2(x,s)u_3(x,s)ds\
      u_2(x,t) &= u_02(x) + int_0^t -P_3(x,s)u_1(x,s) + P_1(x,s)u_3(x,s)ds\
      u_3(x,t) &= u_03(x) + int_0^t P_2(x,s)u_1(x,s) - P_1(x,s)u_2(x,s)ds,
      endalign*

      satisfies the initial value problem involving time derivatives, with $u(cdot, 0) = u_0$, while for $t in (0,T)$ fixed, a solution of the form
      beginalign*
      beginaligned
      u_1(x,t) &= g_1(t) + int_0^x Q_3(xi,t)u_2(xi,t) - Q_2(xi, t)u_3(xi, t)dxi \
      u_2(x,t) &= g_2(t) + int_0^x -Q_3(xi, t)u_1(xi, t) + Q_1(xi, t) u_3(xi, t)dxi\
      u_3(x,t) &= g_3(t) + int_0^x Q_2(xi, t)u_1(xi, t) - Q_1(xi, t)u_2(xi, t)dxi.
      endaligned
      endalign*

      satisfies the initial value problem involving space derivatives, with $u(0, cdot) = g$. I wanted to show, using the four conditions, that both expressions for the solution coincide. Using conditions 3 and 4 we can rewrite both expressions as
      beginalign*
      u_1(x,t) &= u_01(0) + int_0^x Q_3(xi, 0)u_02(xi) - Q_2(xi, 0)u_03(xi)dxi \
      &quad + int_0^t P_3(0,s)g_2(s) - P_2(0,s)g_3(s)ds + int_0^t int_0^x partial_x (P_3 u_2 - P_2 u_3 )(xi, s)dxi ds\
      u_2(x,t) &= u_02(0) + int_0^x -Q_3(xi, 0)u_01 + Q_1(xi, 0)u_03(xi)dxi \
      &quad + int_0^t -P_3(0,s)g_1(s) + P_1(0,s)g_3(s)ds + int_0^t int_0^x partial_x(-P_3 u_1 + P_1 u_3)(xi, s)ds\
      u_3(x,t) &= u_03(0) + int_0^x Q_2(xi, 0)u_01(xi) - Q_1(xi, 0)u_02(xi)\
      &quad + int_0^t P_2(0,s)g_1(s) - P_1(0,s)g_2(s)ds + int_0^t int_0^x partial_x (P_2 u_1 - P_1 u_2)(xi,s)dxi ds,
      endalign*

      and
      beginalign*
      u_1(x,t) &= g_1(0) + int_0^t P_3(0,s)g_2(s) - P_2(0,s)g_3(s) ds \
      &quad + int_0^x Q_3(xi,0)u_02(xi) - Q_2(xi, 0)u_03(xi)dxi + int_0^x int_0^t partial_t ( Q_3 u_2 - Q_2 u_3)(xi, s)dsdxi\
      u_2(x,t) &= g_2(0) + int_0^t -P_3(0,s)g_1(s) + P_1(0,s)g_3(s) ds\
      &quad + int_0^x -Q_3(xi, 0)u_01(xi) + Q_1(xi, 0) u_03(xi)dxi + int_0^x int_0^t partial_t (-Q_3 u_1 + Q_1 u_3)(xi,s)dsdxi\
      u_3(x,t) &= g_3(0) + int_0^t P_2(0,s)g_1(s) - P_1(0,s)g_2(s) ds \
      &quad + int_0^x Q_2(xi, 0)u_01(xi) - Q_1(xi, 0)u_02(xi)dxi + int_0^x int_0^t partial_t ( Q_2 u_1 - Q_1 u_2)(xi, s)ds dxi
      endalign*

      respectively. It seems that both expressions would coincide if the following equalities hold:
      beginalign*
      partial_x (P_3 u_2 - P_2 u_3 ) &= partial_t ( Q_3 u_2 - Q_2 u_3)\
      partial_x(-P_3 u_1 + P_1 u_3) &= partial_t (-Q_3 u_1 + Q_1 u_3)\
      partial_x (P_2 u_1 - P_1 u_2) &= partial_t ( Q_2 u_1 - Q_1 u_2).
      endalign*

      To show that these equalities hold, I would like to use condition 1. However, I do not know how to argue from there.



      Any suggestion of how to proceed from here, any alternative way of reasoning, or explanation of why there would be no solution, would be welcome. Thank you.



      This question is also posed on MathOverflow: https://mathoverflow.net/questions/326280/6-linear-pde-for-only-3-unknowns.







      pde systems-of-equations linear-pde






      share|cite|improve this question















      share|cite|improve this question













      share|cite|improve this question




      share|cite|improve this question








      edited Mar 28 at 8:10







      user344045

















      asked Mar 24 at 14:33









      user344045user344045

      907




      907




















          0






          active

          oldest

          votes












          Your Answer








          StackExchange.ready(function()
          var channelOptions =
          tags: "".split(" "),
          id: "69"
          ;
          initTagRenderer("".split(" "), "".split(" "), channelOptions);

          StackExchange.using("externalEditor", function()
          // Have to fire editor after snippets, if snippets enabled
          if (StackExchange.settings.snippets.snippetsEnabled)
          StackExchange.using("snippets", function()
          createEditor();
          );

          else
          createEditor();

          );

          function createEditor()
          StackExchange.prepareEditor(
          heartbeatType: 'answer',
          autoActivateHeartbeat: false,
          convertImagesToLinks: true,
          noModals: true,
          showLowRepImageUploadWarning: true,
          reputationToPostImages: 10,
          bindNavPrevention: true,
          postfix: "",
          imageUploader:
          brandingHtml: "Powered by u003ca class="icon-imgur-white" href="https://imgur.com/"u003eu003c/au003e",
          contentPolicyHtml: "User contributions licensed under u003ca href="https://creativecommons.org/licenses/by-sa/3.0/"u003ecc by-sa 3.0 with attribution requiredu003c/au003e u003ca href="https://stackoverflow.com/legal/content-policy"u003e(content policy)u003c/au003e",
          allowUrls: true
          ,
          noCode: true, onDemand: true,
          discardSelector: ".discard-answer"
          ,immediatelyShowMarkdownHelp:true
          );



          );













          draft saved

          draft discarded


















          StackExchange.ready(
          function ()
          StackExchange.openid.initPostLogin('.new-post-login', 'https%3a%2f%2fmath.stackexchange.com%2fquestions%2f3160600%2f6-linear-pde-for-only-3-unknowns%23new-answer', 'question_page');

          );

          Post as a guest















          Required, but never shown

























          0






          active

          oldest

          votes








          0






          active

          oldest

          votes









          active

          oldest

          votes






          active

          oldest

          votes















          draft saved

          draft discarded
















































          Thanks for contributing an answer to Mathematics Stack Exchange!


          • Please be sure to answer the question. Provide details and share your research!

          But avoid


          • Asking for help, clarification, or responding to other answers.

          • Making statements based on opinion; back them up with references or personal experience.

          Use MathJax to format equations. MathJax reference.


          To learn more, see our tips on writing great answers.




          draft saved


          draft discarded














          StackExchange.ready(
          function ()
          StackExchange.openid.initPostLogin('.new-post-login', 'https%3a%2f%2fmath.stackexchange.com%2fquestions%2f3160600%2f6-linear-pde-for-only-3-unknowns%23new-answer', 'question_page');

          );

          Post as a guest















          Required, but never shown





















































          Required, but never shown














          Required, but never shown












          Required, but never shown







          Required, but never shown

































          Required, but never shown














          Required, but never shown












          Required, but never shown







          Required, but never shown







          Popular posts from this blog

          Lowndes Grove History Architecture References Navigation menu32°48′6″N 79°57′58″W / 32.80167°N 79.96611°W / 32.80167; -79.9661132°48′6″N 79°57′58″W / 32.80167°N 79.96611°W / 32.80167; -79.9661178002500"National Register Information System"Historic houses of South Carolina"Lowndes Grove""+32° 48' 6.00", −79° 57' 58.00""Lowndes Grove, Charleston County (260 St. Margaret St., Charleston)""Lowndes Grove"The Charleston ExpositionIt Happened in South Carolina"Lowndes Grove (House), Saint Margaret Street & Sixth Avenue, Charleston, Charleston County, SC(Photographs)"Plantations of the Carolina Low Countrye

          random experiment with two different functions on unit interval Announcing the arrival of Valued Associate #679: Cesar Manara Planned maintenance scheduled April 23, 2019 at 00:00UTC (8:00pm US/Eastern)Random variable and probability space notionsRandom Walk with EdgesFinding functions where the increase over a random interval is Poisson distributedNumber of days until dayCan an observed event in fact be of zero probability?Unit random processmodels of coins and uniform distributionHow to get the number of successes given $n$ trials , probability $P$ and a random variable $X$Absorbing Markov chain in a computer. Is “almost every” turned into always convergence in computer executions?Stopped random walk is not uniformly integrable

          How should I support this large drywall patch? Planned maintenance scheduled April 23, 2019 at 00:00UTC (8:00pm US/Eastern) Announcing the arrival of Valued Associate #679: Cesar Manara Unicorn Meta Zoo #1: Why another podcast?How do I cover large gaps in drywall?How do I keep drywall around a patch from crumbling?Can I glue a second layer of drywall?How to patch long strip on drywall?Large drywall patch: how to avoid bulging seams?Drywall Mesh Patch vs. Bulge? To remove or not to remove?How to fix this drywall job?Prep drywall before backsplashWhat's the best way to fix this horrible drywall patch job?Drywall patching using 3M Patch Plus Primer